Proof that the exponential function is convex

Click For Summary
The discussion revolves around proving the convexity of the exponential function, specifically f(x) = e^(ax). Participants highlight that a function is convex if its second derivative, f''(x), is greater than zero for all x in the interval. It is noted that for f(x) = e^(ax), the second derivative f''(x) = a^2 e^(ax) is indeed positive for all real numbers, confirming convexity. There is some debate about the necessity of strict convexity versus general convexity, but the consensus is that the condition f''(x) > 0 is sufficient for establishing convexity. The discussion emphasizes understanding the definitions and implications of convex functions in mathematical analysis.
L Navarro H
Messages
3
Reaction score
0
Homework Statement
f(x)=e^(ax)
where a>0
Relevant Equations
A function f(x) is convex if the statement that is into the question marks proofs
I try to proof it but i got stuck right here, i want your opinions
Can I get a solution if i continue by this way? or Do I have to take another? and if it is so, what would yo do?
 

Attachments

  • 127809042_211031057062186_8356437189153175961_n.jpg
    127809042_211031057062186_8356437189153175961_n.jpg
    45.9 KB · Views: 330
Physics news on Phys.org
Isn't it sufficient to say that ##f(x)## is convex on ##(-\infty, \infty)## if ##f''(x) > 0## for all ##x## in that interval? If ##f(x) = e^{ax}## then ##f''(x) = a^2 e^{ax} > 0, \forall x \in \mathbb{R}##.
 
L Navarro H said:
Relevant Equations:: A function f(x) is convex if the statement that is into the question marks proofs
What does the above mean?
It's hardly an equation, let alone relevant.
 
etotheipi said:
Isn't it sufficient to say that ##f(x)## is convex on ##(-\infty, \infty)## if ##f''(x) > 0## for all ##x## in that interval? If ##f(x) = e^{ax}## then ##f''(x) = a^2 e^{ax} > 0, \forall x \in \mathbb{R}##.
> gives strictly convex, we don't need that
 
pbuk said:
> gives strictly convex, we don't need that

Well I suppose that's true , but I did say 'if' and not 'iff'! So what I wrote is true st`atement 😜
 
Question: A clock's minute hand has length 4 and its hour hand has length 3. What is the distance between the tips at the moment when it is increasing most rapidly?(Putnam Exam Question) Answer: Making assumption that both the hands moves at constant angular velocities, the answer is ## \sqrt{7} .## But don't you think this assumption is somewhat doubtful and wrong?

Similar threads

Replies
7
Views
2K
  • · Replies 6 ·
Replies
6
Views
2K
  • · Replies 5 ·
Replies
5
Views
2K
  • · Replies 3 ·
Replies
3
Views
1K
Replies
8
Views
2K
Replies
1
Views
2K
  • · Replies 9 ·
Replies
9
Views
2K
  • · Replies 14 ·
Replies
14
Views
2K
Replies
1
Views
2K
  • · Replies 24 ·
Replies
24
Views
4K